Author Topic: Woodward's effect  (Read 803005 times)

Offline birchoff

  • Full Member
  • **
  • Posts: 273
  • United States
  • Liked: 125
  • Likes Given: 95
Re: Woodward's effect
« Reply #740 on: 05/09/2016 10:47 pm »
Hallo wallofwolfstreet,

can you comment on this paper?

Thanks.

Sure thing.  Unfortunately the pdf has content copying not allowed, so I can’t just copy and paste.  To save myself time I’ll just offer the first little bit of a section I want to quote and you can ctrl+f it.  Apologies in advance for any mistakes in copying. 

In general, the paper offers  a result that is completely consistent with the Oak Ridge Scientists (ORS) interpretation of the correct form of the equation of motion for a variable mass system (John Cramer of the University of Washington also advances this equation of motion, see the second paragraph of the intro of this 2004 paper).

Right in the abstract, Wanser writes:

Quote
There is no rocket type thrust in the usual sense of ejecting propellant, since it is supposed that there is no relative velocity along the direction of motion associated with the mass changes.

Emphasis mine.

This assumption of zero relative velocity is critical, because it allows him to present F=ma+dmdtvrel as just F=ma (i.e. he has reduced the general equation of motion to the Woodward's equation of motion under the constraint that vrel=0).  To be honest, I kind of feel that this paper is presenting a bit of a trivial result.  It is simply showing that when the mass flow into and out of a variable mass system is contrived so that there is no net relative velocity, then the center of mass of the open system can be made to accelerate.  This is basically what Mezzenile has said in his preceding posts.  The issue is, in order for the mass that flows in and out of the system to have vrel=0 despite the acceleration of the MET, there must be some net force accelerating this variable mass, which Wanser correctly identifies in his Discussion:

(start reading at ‘the center of mass of the cart’)

Quote

This momentum must be made up by the rest of the universe allowing the isotropically ejected mass (in the rest frame of the accelerating body) to return isotropically to the body in its rest frame, that has in the mean time accelerated forward, to keep pace with the MET device, thus requiring the “spherical shell  of ejected mass to move forward.

Wanser also reiterates that the results are contingent on assuming vrel=0.  It’s this assumption that doesn’t have any theoretical motivation as being applicable to METs as far as I can see, and in fact Woodward routinely advances explanations as to how you can set vrel=0 which just aren’t consistent with physics.

Quote
The unidirectional acceleration found here is a consequence of our primary assumption, the possibility of changing the mass of an isolated system with zero relative velocity associated with the net convective momentum flux which produces the changing mass


Essentially, what Wanser has done is what I consider to be a bit of slight of hand (at least in regard to METs).  He has moved the problem from within a control volume containing the MET to outside the control volume.  The only way the variable mass can consistently have vrel=0 despite the claimed acceleration of the MET is that somehow the variable mass which exists outside the control volume is being accelerated by some unknown force alongside the MET.  The question is simply reframed from “what causes the MET to accelerate?” to “what causes the variable mass outside the system boundary of the MET to accelerate with the MET, so the net velocity of this mass is always just right to ignore vrel dependent terms?”

So like I said, I think this result is a bit trivial, because it just flips the problem definition around, demonstrating that it is possible to self-accelerate a variable mass system if you are somehow able to accelerate the variable mass when it is outside your control volume boundary.  I don’t think that's a very surprising result. 

There are a few things in the paper that I disagree with (Newton’s third law more fundamental that Conservation of Momentum? – I don’t think so), but I don’t think they are particularity relevant to the question at hand.

So in short. your calling into question the explaination used to argue that the mass change said to affect the FM component of a MET device can be done in a manner that is consistent with known physics?

Offline wallofwolfstreet

  • Full Member
  • *
  • Posts: 165
  • Liked: 169
  • Likes Given: 436
Re: Woodward's effect
« Reply #741 on: 05/10/2016 12:09 am »
So in short. your calling into question the explaination used to argue that the mass change said to affect the FM component of a MET device can be done in a manner that is consistent with known physics?

In short, Yes.

This is basically my read on Mach effects and Woodward's work:

There are three Mach effects proposed by Woodward.  The first Mach effect (also called the Woodward effect) is proposed to cause transient mass fluctuations.  It is then proposed that by fluctuating a mass (the FM component of an MET) in phase with a cyclical displacement one can generate acceleration of a closed system.  It is this second proposal that lacks a coherent derivation in my opinion.  I honestly don't have the theoretical grounding in general relativity or the work of Sciama to either confirm or deny whether the first proposal has a valid derivation.

Transient mass fluctuations may or may not be real, but transient mass fluctuations do not necessarily imply METs.

I'm not the first person to take this view either.  It is essentially the view taken by Cramer et al. (2004) in Test of Mach's Principle with a Mechanical Oscillator

Quote
Unfortunately, this scheme for observing the
inertia variation appears to be at odds with the
relativistically invariant form of Newton's 2nd law of
motion:

F = dp/dt = mdv/dt + vdm/dt                              (1)

Since the inertial mass m of the test body is
expected to vary with time, the last term of Equation
(1) cannot be ignored. It is not surprising, in view of
Newton's 3rd law of motion, that for any sinusoidal
variations of the mass around a central value, the force
contribution from the v dm/dt term is found to
precisely cancel the supposed "unbalanced force"
arising from the m dv/dt term, leading to a time averaged
net force of zero on the overall system.
From this simple calculation, it appears that
unbalanced force searches are not good tests of the
proposed effect.

There remains the question of
whether the Woodward inertia variation is present in a
system having an energy flow. In the present work, we
have devised two tests of this effect that do not require the
presence of an unbalanced inertial force.
« Last Edit: 05/10/2016 12:11 am by wallofwolfstreet »

Offline birchoff

  • Full Member
  • **
  • Posts: 273
  • United States
  • Liked: 125
  • Likes Given: 95
Re: Woodward's effect
« Reply #742 on: 05/10/2016 06:05 am »
So in short. your calling into question the explaination used to argue that the mass change said to affect the FM component of a MET device can be done in a manner that is consistent with known physics?

In short, Yes.

This is basically my read on Mach effects and Woodward's work:

There are three Mach effects proposed by Woodward.  The first Mach effect (also called the Woodward effect) is proposed to cause transient mass fluctuations.  It is then proposed that by fluctuating a mass (the FM component of an MET) in phase with a cyclical displacement one can generate acceleration of a closed system.  It is this second proposal that lacks a coherent derivation in my opinion.  I honestly don't have the theoretical grounding in general relativity or the work of Sciama to either confirm or deny whether the first proposal has a valid derivation.

Transient mass fluctuations may or may not be real, but transient mass fluctuations do not necessarily imply METs.

I'm not the first person to take this view either.  It is essentially the view taken by Cramer et al. (2004) in Test of Mach's Principle with a Mechanical Oscillator

Quote
Unfortunately, this scheme for observing the
inertia variation appears to be at odds with the
relativistically invariant form of Newton's 2nd law of
motion:

F = dp/dt = mdv/dt + vdm/dt                              (1)

Since the inertial mass m of the test body is
expected to vary with time, the last term of Equation
(1) cannot be ignored. It is not surprising, in view of
Newton's 3rd law of motion, that for any sinusoidal
variations of the mass around a central value, the force
contribution from the v dm/dt term is found to
precisely cancel the supposed "unbalanced force"
arising from the m dv/dt term, leading to a time averaged
net force of zero on the overall system.
From this simple calculation, it appears that
unbalanced force searches are not good tests of the
proposed effect.

There remains the question of
whether the Woodward inertia variation is present in a
system having an energy flow. In the present work, we
have devised two tests of this effect that do not require the
presence of an unbalanced inertial force.

Thank you for clarifying that. In that case your critique of the paper Flux Capacitor posted a few posts back, in light of your your comments on the Wasner paper that mubahni posted to the forum; Is either misdirected, wrong, or I am simply reading more into it than you intended. Since that paper explains pretty much the same thing that the wasner paper does, with one difference that I think is a minor one. It does not explicitly state that it is making the assumption that it is possible to create a system where one can change the mass of a component in just such a way that it is possible to zero out the vdm/dt term. The paper in question basically assumes such a thing is possible and talks about an example of such a system. The Wasner paper goes a bit further on the analysis of why such a thing doesnt violate newtons second law, and takes as an assumption that the Mach Effect demonstrated in the MET device is capable of changing the mass in the FM component as described.

Now as for your real critique. I think Woodward and Fearn have put alot of theoretical effort into trying to show that the transient mass change mach effect is real. Part 1 of Making Starships and Stargates is intended to be a narration at an engineers level of how the two Mach Effects are derived from GRT, the first one being the transient mass change. At the end of Chapter 3 (the last chapter in part 1), Addendum #2 covers the step by step derivation of the Mach effect terms discussed in Chapter 3. Where Addendum #2 is sourced directly from his published paper "Flux Capacitors and the Origin of Inertia" which you can find on his faculty page.

As for being able to turn that transient mass change into motion. i would agree that their isnt much theoretical work about why this would work. From what I have read it looks like the idea is simply that we can piggy back off of the equal and opposite inertial force applied to the fluctuating mass. So if we push on the FM while it is larger we should get a large inertial force applied to the actuator doing the pushing. If we pull on the FM when it is smaller we should get a smaller inertial force. This should yield a net unidirectional force. So from where I stand, its no the generation of the uni directional force that requires proof and eventual acceptance its the transient mass change. If you can transiently change the mass of the FM component of a MET in the manner described in the Wasner paper then generating a uni directional force from such a scheme is the easy part.

So I would definately ask. What is missing from the work Woodward and Fern has done so far?

N.B. Where did you see there were three Mach Effects. Woodward has only ever talked about two Mach Effects. First one is the transient mass change (impulse) term, and the second one is referred to as the Worm Hole term.

Offline birchoff

  • Full Member
  • **
  • Posts: 273
  • United States
  • Liked: 125
  • Likes Given: 95
Re: Woodward's effect
« Reply #743 on: 05/10/2016 05:17 pm »
...

In short, Yes.

This is basically my read on Mach effects and Woodward's work:

There are three Mach effects proposed by Woodward.  The first Mach effect (also called the Woodward effect) is proposed to cause transient mass fluctuations.  It is then proposed that by fluctuating a mass (the FM component of an MET) in phase with a cyclical displacement one can generate acceleration of a closed system.  It is this second proposal that lacks a coherent derivation in my opinion.  I honestly don't have the theoretical grounding in general relativity or the work of Sciama to either confirm or deny whether the first proposal has a valid derivation.

Transient mass fluctuations may or may not be real, but transient mass fluctuations do not necessarily imply METs.

I'm not the first person to take this view either.  It is essentially the view taken by Cramer et al. (2004) in Test of Mach's Principle with a Mechanical Oscillator

Quote
Unfortunately, this scheme for observing the
inertia variation appears to be at odds with the
relativistically invariant form of Newton's 2nd law of
motion:

F = dp/dt = mdv/dt + vdm/dt                              (1)

Since the inertial mass m of the test body is
expected to vary with time, the last term of Equation
(1) cannot be ignored. It is not surprising, in view of
Newton's 3rd law of motion, that for any sinusoidal
variations of the mass around a central value, the force
contribution from the v dm/dt term is found to
precisely cancel the supposed "unbalanced force"
arising from the m dv/dt term, leading to a time averaged
net force of zero on the overall system.
From this simple calculation, it appears that
unbalanced force searches are not good tests of the
proposed effect.

There remains the question of
whether the Woodward inertia variation is present in a
system having an energy flow. In the present work, we
have devised two tests of this effect that do not require the
presence of an unbalanced inertial force.

So now that i have a little sleep in me. Something else has occured to me about your critique. Why assert that cramer takes the perspective that you quoted. When in the Wasner paper, that you seem to agree with, it is said that Cramer incorrectly includes the vdm/dt term because it the transient mass change as far as the wasner paper is concerned is the type of mass change that allows you to set that term to 0. What I would expect to see in the Cramer paper is some exposition on why the transient mass change term in woodwards derivation would not lead to the type of mass change that allows you to set the vdm/dt term to 0.

I think what gets on my nerves most about this criticism is that both sides seem to be talking past each other. Woodward pretty much believes that the theoretical work he has done up till now justifies that the transient mass change is such that you are allowed to set the vdm/dt term to 0. While criticisms I have seen so far amount to saying "YOU CAN NEVER SET THE vdm/dt TERM to ZERO". Even though the Wasner paper outlines a thorough explaination of when you are allowed to set the vdm/dt term to zero. So either the Wasner paper is pure rubbish, or it isnt and there are specific situations where you are allowed to do what woodward has done. If the later is true then the only valid criticism should be centered on his derivation of the transient mass change and showing how it doesnt lead to a situation where it is correct to set the vdm/dt term to zero.

Personally I think the ball is in the critics court mostly because Woodward has laid out his step by step derivations in published work and provided alot of exposition explaining his reasoning. In addition Fearn's recent work has  allowed them to use Hoyle and Narlikar's Theory of Gravitation as a starting point for the mach effect derivation, after updating it to handle the divergence issue Hawking raised. This doesnt mean the theory is right. I dont think Woodward believes the theory is completely right.  Primarily because the predicted force magnitudes are way higher than they have been able to show with experiments. However, the HN Theory work Fearn did seems to have corrected the derivation enough to make better predictions about how thrust could scale.

Anyway. now that I understand what your criticism is. I think the focus on newtons equation of motion is misplaced. The real focus as I see it should be on why Woodward's transient mass change term does not produce an effect that allows the vdm/dt term of the equation of motion to be set to 0.

Offline wallofwolfstreet

  • Full Member
  • *
  • Posts: 165
  • Liked: 169
  • Likes Given: 436
Re: Woodward's effect
« Reply #744 on: 05/11/2016 02:19 am »
Thank you for clarifying that. In that case your critique of the paper Flux Capacitor posted a few posts back, in light of your your comments on the Wasner paper that mubahni posted to the forum; Is either misdirected, wrong, or I am simply reading more into it than you intended.

Please quote exactly what you feel is wrong, misdirected, or what you want me to clarify from my original comments:
https://forum.nasaspaceflight.com/index.php?topic=31037.msg1531414#msg1531414
https://forum.nasaspaceflight.com/index.php?topic=31037.msg1531664#msg1531664   

Quote
Since that paper explains pretty much the same thing that the wasner paper does, with one difference that I think is a minor one.

The papers are very much different. The Wasner paper and the ORNL response paper (what flux capacitor just posted; attached as Woodward answer to ORNL.pdf) are not the same thing.  The ORNL response paper has a basic physics error in it; the entire experiment Woodward is claiming supports his equation of motion over the correct ORS equation of motion is complete nonsense because you can’t discern the difference between the two equations of motion from the experiment.   You have to understand, the entire meat of that paper, namely the draining water bottle on a track experiment, is completely botched as I demonstrated in my original response.  Figure 4, which is supposed to demonstrate the victory of F=ma over F=ma + vdm/dt, is wrong; the red line should be coincident with the blue line (and the measured data points).  If Woodward wants a basic mechanics experiment to determine the correct equation of motion, he should try the experiment I suggested:
Quote
One way to demonstrate why Woodward’s equation of motion is wrong is to consider a flatcar rolling down a smooth level track at constant velocity, where a hopper from above is pouring sand onto the cart.  This is the exact inverse of Woodward’s thought experiment, so Woodward’s equation of motion, F=m(t)dv/dt, should be applicable

My question is this:  Does the flatcar in my example accelerate, or continue to travel at constant velocity when the sand is being poured onto the flatcar?

So people who read Woodward’s rebuttal and actually thought it made sense, please use Woodward’s equation of motion in this instance to make a prediction: does the flatcar slow down or not?     

What does the Woodward equation of motion predict, what does the ORS equation of motion predict, and what actually happens?

I know I seem a bit condensing here, but I didn’t mean the above questions to be rhetorical.  I’d actually appreciate an answer to those questions, because the answer to that question lets you know whether to toss out the ORNL response paper or not.  Answer those questions correctly, and you will see I went easy on the ORNL response paper. 

Quote
It does not explicitly state that it is making the assumption that it is possible to create a system where one can change the mass of a component in just such a way that it is possible to zero out the vdm/dt term. The paper in question basically assumes such a thing is possible and talks about an example of such a system.

Excerpt for the fact that you've completely omitted the experimental part?  And if the assumption isn't explicity stated, then how so you know it's been made at all?  And if Woodward is assuming that vrel = 0, how does that answer anything?  He has to prove it, not assume it, for anyone to care. 

Quote
 
The Wasner paper goes a bit further on the analysis of why such a thing doesnt violate newtons second law, and takes as an assumption that the Mach Effect demonstrated in the MET device is capable of changing the mass in the FM component as described.

Correct, assumption is the critical word here.  That's why I accused the Wasner paper of being trivial.  It's not hard to show the self acceleration of a control volume if you assume mass just shows up with no relative velocity.  That's the rocket equivalent of attaching a fuel hose to your rocket so that as it flies, more propellant just shows up in the fuel tank without altering the momentum of the rocket.  It's not unexpected that the rocket now self accelerates when you ignore the action exterior to the rocket (which is what Wanser does), namely the need of accelerating the propellant to get it to the rocket in the first place. 

Quote
So I would definately ask. What is missing from the work Woodward and Fern has done so far?

What is missing is that I have yet to see a coherent explanation that validates dropping vdm/dt.  Without that explanation, the entire theory of METs is foundation less.  I’m really just echoing Whealton et al. and Cramer et al. when I say this, who were evidently as unimpressed by Woodward’s response as I am seeing as how they never pursued METs further.  In fact, see the attached ORNL presentation, slides 9,24,25.

Quote
N.B. Where did you see there were three Mach Effects. Woodward has only ever talked about two Mach Effects. First one is the transient mass change (impulse) term, and the second one is referred to as the Worm Hole term.

https://en.wikipedia.org/wiki/Woodward_effect

Quote from: wikipedia
The Woodward effect, also referred to as a Mach effect, one of at least three predicted Mach effects

According to Woodward, at least three Mach effects are theoretically possible: vectored impulse thrust, open curvature of spacetime, and closed curvature of spacetime

So now that i have a little sleep in me. Something else has occured to me about your critique. Why assert that cramer takes the perspective that you quoted. When in the Wasner paper, that you seem to agree with, it is said that Cramer incorrectly includes the vdm/dt term because it the transient mass change as far as the wasner paper is concerned is the type of mass change that allows you to set that term to 0. What I would expect to see in the Cramer paper is some exposition on why the transient mass change term in woodwards derivation would not lead to the type of mass change that allows you to set the vdm/dt term to 0.

Nothing in the known universe lets you set the vdm/dt term to 0 globally for an object that is accelerating, and hence has infinitely many different possible velocities in a single reference frame.  That’s the whole point.  It’s not Cramers job to show that METs obey established physics, it’s Woodward’s job to show they don’t.

Quote
I think what gets on my nerves most about this criticism is that both sides seem to be talking past each other. Woodward pretty much believes that the theoretical work he has done up till now justifies that the transient mass change is such that you are allowed to set the vdm/dt term to 0. While criticisms I have seen so far amount to saying "YOU CAN NEVER SET THE vdm/dt TERM to ZERO".

EXACTLY.  That’s literally the whole criticism for a device which claims to accelerate.  Woodward’s supposed response to that criticism, as contained in the ORNL response paper, is complete rubbish because it cocks up a physics 101 experiment.  So here we are back to square one.

Quote
Even though the Wasner paper outlines a thorough explaination of when you are allowed to set the vdm/dt term to zero.

No it doesn't, it assumes that you can set it equal to zero (you yourself just said this a few quotes up).  It doesn’t validate the belief that you can set it to zero, and I don't see any explanation of when you are allowed to set it to zero.  It’s more or less an assumption of the paper; there is no theoretical justification for it.

Quote
So either the Wasner paper is pure rubbish, or it isnt and there are specific situations where you are allowed to do what woodward has done.

The Wasner paper isn’t pure rubbish because it doesn’t try to sweep the all important assumption under the rug.  That’s the critical difference.   Right from the Wasner paper:

Quote from: Wasner
We can not have any net ejection of mass in a cyclic process, so this condition is satisfied if, we assume that mass fluctuations are of this type, i.e. that vrel=0

Quote
Personally I think the ball is in the critics court mostly because Woodward has laid out his step by step derivations in published work and provided alot of exposition explaining his reasoning.

He has provided alot of exposition.  The problem is, at least in the case of the ORNL response, is that the exposition and reasoning is completely wrong.  I went through point by point in that paper and showed the flaws, and the sand falling onto the cart problem clarifies why Woodward is wrong and ORNL are right.

Quote
Anyway. now that I understand what your criticism is. I think the focus on newtons equation of motion is misplaced. The real focus as I see it should be on why Woodward's transient mass change term does not produce an effect that allows the vdm/dt term of the equation of motion to be set to 0.

Well, no.  Like I mentioned above, every single phenomena known to man “does not produce an effect that allows the vdm/dt term of the equation of motion to be [universally] set to zero [when the control volume in question is claimed to accelerate] ”.  If Woodward wants to claim that METs break out of that paradigm sure, but the onus is purely on him to back up that claim.   
« Last Edit: 05/11/2016 02:22 am by wallofwolfstreet »

Offline Mezzenile

  • Full Member
  • *
  • Posts: 130
  • Liked: 63
  • Likes Given: 24
Re: Woodward's effect
« Reply #745 on: 05/19/2016 08:27 am »
I discover the here above cited article from Cramer et al. (2004) : Test of Mach's Principle with a Mechanical Oscillator

It is a pity to see that even seasoned professionals occasionally make mistakes when applying the second law, even in simple circumstances :

The first equation of the article (F = dp/dt = mdv/dt + vdm/dt) is introduced as beeing the relativistically  invariant  form  of  Newton's  2nd law  of motion.

But this formulation is FALSE. The term vdm/dt is completely wrong ! It should be replaced by udm/dt where u is a speed that all observers can agree on and so which has an invariant meaning – for example, the velocity of the just ejected exhaust plume of a rocket with respect to the rocket ( and not the speed v of the rocket !).

Apparently the three distinguished authors of the article : John G. Cramer , Curran W. Fey, and Damon V. Casissi are not yet fully convinced that rockets can work !!!  :D :D




Offline wallofwolfstreet

  • Full Member
  • *
  • Posts: 165
  • Liked: 169
  • Likes Given: 436
Re: Woodward's effect
« Reply #746 on: 05/22/2016 04:17 am »
I discover the here above cited article from Cramer et al. (2004) : Test of Mach's Principle with a Mechanical Oscillator

It is a pity to see that even seasoned professionals occasionally make mistakes when applying the second law, even in simple circumstances :

The first equation of the article (F = dp/dt = mdv/dt + vdm/dt) is introduced as beeing the relativistically  invariant  form  of  Newton's  2nd law  of motion.

But this formulation is FALSE. The term vdm/dt is completely wrong ! It should be replaced by udm/dt where u is a speed that all observers can agree on and so which has an invariant meaning – for example, the velocity of the just ejected exhaust plume of a rocket with respect to the rocket ( and not the speed v of the rocket !).

Apparently the three distinguished authors of the article : John G. Cramer , Curran W. Fey, and Damon V. Casissi are not yet fully convinced that rockets can work !!!  :D :D

True, it would have been better for them to have been more rigorous and use the correct expression with relative velocity.  Goes to show how easy it is to get tripped up.  This trip up reduces the generality of Equation (7), but it doesn't adversely affect the paper as a whole. 
« Last Edit: 05/22/2016 04:26 am by wallofwolfstreet »

Offline Mezzenile

  • Full Member
  • *
  • Posts: 130
  • Liked: 63
  • Likes Given: 24
Re: Woodward's effect
« Reply #747 on: 05/22/2016 12:47 pm »
True, it would have been better for them to have been more rigorous and use the correct expression with relative velocity.  Goes to show how easy it is to get tripped up.  This trip up reduces the generality of Equation (7), but it doesn't adversely affect the paper as a whole.

It is not only a question to be more rigorous and to use the correct expression. It challenges also the main thesis of the article which relies on this false equation together with what the authors call a “simple calculation” to support the claim that the searches of unbalanced force are not good tests of the Woodward effect.

This beeing said does somebody know if the authors finally managed to get rid of the the spurious excitation which perturbated their measurements ?
« Last Edit: 05/22/2016 12:52 pm by Mezzenile »

Offline birchoff

  • Full Member
  • **
  • Posts: 273
  • United States
  • Liked: 125
  • Likes Given: 95
Re: Woodward's effect
« Reply #748 on: 05/23/2016 11:23 pm »
True, it would have been better for them to have been more rigorous and use the correct expression with relative velocity.  Goes to show how easy it is to get tripped up.  This trip up reduces the generality of Equation (7), but it doesn't adversely affect the paper as a whole.

It is not only a question to be more rigorous and to use the correct expression. It challenges also the main thesis of the article which relies on this false equation together with what the authors call a “simple calculation” to support the claim that the searches of unbalanced force are not good tests of the Woodward effect.

This beeing said does somebody know if the authors finally managed to get rid of the the spurious excitation which perturbated their measurements ?

I see your point. but after my discussion with wallofwolfstreet. I am beginning to understand that the core problem here isnt so much what that term in the equation of motion means. Its whether or not the effect being displayed is allowed to zero out that term in the equation of motion.

So the million dollar question remains, where in the work woodward et al have done does it show that the transient mass fluctuation mach effect is just such an effect that allows you to zero out that term in the motion equation.

Personally I think the answer to that question is the experimental evidence that woodward is building. Their theory argues that this particular mach effect should work this way. Now they are on to proving that their experiments match their theoretical predictions.

Offline wallofwolfstreet

  • Full Member
  • *
  • Posts: 165
  • Liked: 169
  • Likes Given: 436
Re: Woodward's effect
« Reply #749 on: 05/24/2016 03:26 pm »
I see your point. but after my discussion with wallofwolfstreet. I am beginning to understand that the core problem here isnt so much what that term in the equation of motion means. Its whether or not the effect being displayed is allowed to zero out that term in the equation of motion.

Exactly.  In the case where we use the correct  F = dp/dt = mdv/dt + udm/dt, you just substitute u=w-v, where w is the velocity of the mass flow in the frame of reference in which v is measured.  You still end up with the exact same probelm, namely "How can the velocity term in the full equation of motion F=dp/dt be zero despite the fact the MET is accelerating?", only now the question has shifted to why should w=0?   

Quote
 
So the million dollar question remains, where in the work woodward et al have done does it show that the transient mass fluctuation mach effect is just such an effect that allows you to zero out that term in the motion equation.

I agree.  I hope I was able to show in my previous post that Woodward's previous attempt at showing this (i.e. the ORNL repsonse paper posted by FluxCapacitor and attached here) is wrong.  Woodward should consider trying to readdress this issue since it is critical for being able to claim propulsion applications from Mass fluctuations

This beeing said does somebody know if the authors finally managed to get rid of the the spurious excitation which perturbated their measurements ?

I don't know what ultimately came of the experiment; it's an interesting question.   

Offline Mezzenile

  • Full Member
  • *
  • Posts: 130
  • Liked: 63
  • Likes Given: 24
Re: Woodward's effect
« Reply #750 on: 05/28/2016 05:11 am »
True, it would have been better for them to have been more rigorous and use the correct expression with relative velocity.  Goes to show how easy it is to get tripped up.  This trip up reduces the generality of Equation (7), but it doesn't adversely affect the paper as a whole.

It is not only a question to be more rigorous and to use the correct expression. It challenges also the main thesis of the article which relies on this false equation together with what the authors call a “simple calculation” to support the claim that the searches of unbalanced force are not good tests of the Woodward effect.

This beeing said does somebody know if the authors finally managed to get rid of the the spurious excitation which perturbated their measurements ?

I see your point. but after my discussion with wallofwolfstreet. I am beginning to understand that the core problem here isnt so much what that term in the equation of motion means. Its whether or not the effect being displayed is allowed to zero out that term in the equation of motion.

So the million dollar question remains, where in the work woodward et al have done does it show that the transient mass fluctuation mach effect is just such an effect that allows you to zero out that term in the motion equation.

Personally I think the answer to that question is the experimental evidence that woodward is building. Their theory argues that this particular mach effect should work this way. Now they are on to proving that their experiments match their theoretical predictions.
Good luck  ;) ;) if you want to understand the origin of inertia by Mach principle and associated mass fluctuation as proposed by Woodward using only the tool of newtonian mechanics. You condemn yourself to stay in the interior of a circle from which you will never escape. If for you an exchange of momentum can only be represented by a mass m moving at a classical speed u you will have no chance to grasp the key corner of momentum flux vehiculed by the gravinertial waves both to the future and from the future and which propagates most probably at the speed of the light.

The roots of Woodward insight come from the Hoyle-Narlikar (HN) theory (which leads to General Relativity in the presence of an event horizon). The best would be to calculate the dynamics of the Woodward test set-up and the associated emerging force, from HN-theory and its first principles using the Euler Lagrange equation. This derivation is a work in progress as announced by H. Fearn, J. F. Woodward and N. van Rossum.

If you want to stick to Newtonian mechanics in an an absolute space, you should simply consider the isotropy of u or in somewhat equivalent form, a null value for it.

Of course, as you write, the experimental evidence that Woodward is building gives confidence in the capability of HN theory to confirm by a theoretical frame the experimental results.
« Last Edit: 05/28/2016 02:50 pm by Mezzenile »

Offline mubahni

  • Member
  • Posts: 3
  • Stuttgart, Germany
  • Liked: 0
  • Likes Given: 0
Re: Woodward's effect
« Reply #751 on: 06/01/2016 06:33 pm »
In this video, Nembo Buldrini seems to talk and even present results of a replication. I say seem, because I don't understand nor speak italian. Watch from the 54th minute.



Offline Mezzenile

  • Full Member
  • *
  • Posts: 130
  • Liked: 63
  • Likes Given: 24
Re: Woodward's effect
« Reply #752 on: 06/02/2016 07:55 pm »
In this video, Nembo Buldrini seems to talk and even present results of a replication...
Nembo Buldrini is a long time collaborator of Woodward and the illustrator of his book  "Making Starships and Stargates".

Offline Povel

  • Member
  • Posts: 89
  • Liked: 49
  • Likes Given: 18
Re: Woodward's effect
« Reply #753 on: 09/03/2016 12:16 pm »
In the video Buldrini is presenting results for a validation of a device provided by Woodward, it's not a replica.

He shows how the results he obtained are compatible to those claimed by W. : the thrust profile in both cases looks similar, with a peak at the start and at the end.
One difference though is the order of magnitude of the thrust observed, Buldrini test gave as a result a thrust about 10 times lower.
The reason of this could be, in his opinion, either the electronic component used for coupling the source of power with the device, because output hasn't been measured directly and the piece is different from the one used by W.,
or a wrong calibration of the thrust balances. This two reasons aren't mutually exclusive.

These results haven't been published on any paper because of these defects regarding validation of the device, but he said that these preliminary results are much stronger than those that he obtained many years ago, when he tested the MLT, because a clear thrust signal is very noticeable.
He added that more complete tests could happen during 2016


Is there any news from Woodward and Fearn?

Offline birchoff

  • Full Member
  • **
  • Posts: 273
  • United States
  • Liked: 125
  • Likes Given: 95
Re: Woodward's effect
« Reply #754 on: 09/25/2016 01:40 pm »
Dr. Rodal,

In light of the presentation you recently did on HN Theory. I would love to know what your take is on where Woodward and Fearn are as far as having a theory that can be used to describe Mach Effects. To the best of my knowledge the last step needed for then to be able to confidently claim their theory is accurate is being able to derive force equations from it that accurately predict thrust. Which means they need to build better MET devices to test thrust scaling as predicted. Are there other things Heidi and Woodward need to do that we are not aware of?

Offline Rodal

  • Senior Member
  • *****
  • Posts: 5911
  • USA
  • Liked: 6124
  • Likes Given: 5564
Re: Woodward's effect
« Reply #755 on: 09/25/2016 01:59 pm »
Dr. Rodal,

In light of the presentation you recently did on HN Theory. I would love to know what your take is on where Woodward and Fearn are as far as having a theory that can be used to describe Mach Effects. To the best of my knowledge the last step needed for then to be able to confidently claim their theory is accurate is being able to derive force equations from it that accurately predict thrust. Which means they need to build better MET devices to test thrust scaling as predicted. Are there other things Heidi and Woodward need to do that we are not aware of?
1. I presented an exact solution for the force (under suitable, well specified assumptions) and its comparison with experiments.  Prior to the workshop, Prof. Hearn actually independently checked the validity of my solution (which comprises hundreds of terms) by hand !

2. The solution predicts the optimal mass for the tail mass section, in agreement with experiments.  The solution correctly predicts the direction of the force.  The solution predicts the extreme dependence of the amplitude of the force with other parameters in the vicinity of resonance.  The solution also predicts other effects.

3. The solution shows various ways on how it may be possible to increase the force from what has been demonstrated in experiments up to now.

4.  In addition, at the Estes Park Breakthrough Propulsion workshop, a physicist that works with General Relativity presented another derivation of Woodward's equation based on linearization of Einstein's General Relativity. 

5.  In addition, at the Estes Park Breakthrough Propulsion workshop independent experimental measurements of the Mach/Sciama/Wooward effect force were presented from 3 other researchers, in 2 separate continents.

-----

 You will be able to watch my presentation (as well as other presentations) at the Space Studies Institute website: www.ssi.org and ensuing discussion with the participants.
« Last Edit: 09/25/2016 02:30 pm by Rodal »

Offline birchoff

  • Full Member
  • **
  • Posts: 273
  • United States
  • Liked: 125
  • Likes Given: 95
Re: Woodward's effect
« Reply #756 on: 09/25/2016 03:06 pm »
Dr. Rodal,

In light of the presentation you recently did on HN Theory. I would love to know what your take is on where Woodward and Fearn are as far as having a theory that can be used to describe Mach Effects. To the best of my knowledge the last step needed for then to be able to confidently claim their theory is accurate is being able to derive force equations from it that accurately predict thrust. Which means they need to build better MET devices to test thrust scaling as predicted. Are there other things Heidi and Woodward need to do that we are not aware of?
1. I presented an exact solution for the force (under suitable, well specified assumptions) and its comparison with experiments.  Prior to the workshop, Prof. Hearn actually independently checked the validity of my solution (which comprises hundreds of terms) by hand !

2. The solution predicts the optimal mass for the tail mass section, in agreement with experiments.  The solution correctly predicts the direction of the force.  The solution predicts the extreme dependence of the amplitude of the force with other parameters in the vicinity of resonance.  The solution also predicts other effects.

3. The solution shows various ways on how it may be possible to increase the force from what has been demonstrated in experiments up to now.

4.  In addition, at the Estes Park Breakthrough Propulsion workshop, a physicist that works with General Relativity presented another derivation of Woodward's equation based on linearization of Einstein's General Relativity. 

5.  In addition, at the Estes Park Breakthrough Propulsion workshop independent experimental measurements of the Mach/Sciama/Wooward effect force were presented from 3 other researchers, in 2 separate continents.

-----

 You will be able to watch my presentation (as well as other presentations) at the Space Studies Institute website: www.ssi.org and ensuing discussion with the participants.

So I am going to beg your forgiveness for what I am about to do...


INNER SPACE GEEK > IT WORKS!!!!!!?
Translation > Given the strong theoretical support. Are you saying that the only thing left is engineering?

Also, I am curious to know if you spent anytime reflecting on the possibility for Mach Effects to "POTENTIALLY" allow the creation of worn holes. I know unlike the thrust term. The Worm hole creation part still has theoretical rough spots. In particular the use of the ADM model of the electron. However, if you have I was curious about your take on that part of Woodward's derivation of Mach Effects.

Online meberbs

  • Senior Member
  • *****
  • Posts: 3096
  • Liked: 3379
  • Likes Given: 777
Re: Woodward's effect
« Reply #757 on: 09/25/2016 03:42 pm »
I have a couple questions about this solution. I have not gotten around to studying general relativity in detail, so I can't directly critique the calculations, but there are a few implications this brings up.

Are these calculations based on general relativity as it is generally known, or are there some tweaks/ additional assumptions?

As I understand it, the explanation for energy/momentum balance is remote interaction with the other objects in the universe. There are 2 ways I can understand this:
1. Instantaneous interaction with remote objects - this would cause problems with causality.
2. Gravitational waves, which as I understand them have the same Energy/momentum constraints as photon thrusters.

I could see the energy/momentum ratio for gravity waves being modified if you don't take the weak-field limit, but that brings up how any non-planet sized device could be outside the weak field limit.

On a related note, considering the size of the constant G, even if the device was only as efficient for propulsion as a photon thruster, being able control and put significant energy energy into gravitational waves could open up all sorts of interesting applications.

Does the theory really predict meaningful magnitude of the forces, or is it just far enough to say that there is a force in a given direction? From what you said I believe it is the former, but I want to check since I know this could be a very hard calculation to do.

Offline Star One

  • Senior Member
  • *****
  • Posts: 13997
  • UK
  • Liked: 3974
  • Likes Given: 220
Re: Woodward's effect
« Reply #758 on: 09/25/2016 03:43 pm »
Dr. Rodal,

In light of the presentation you recently did on HN Theory. I would love to know what your take is on where Woodward and Fearn are as far as having a theory that can be used to describe Mach Effects. To the best of my knowledge the last step needed for then to be able to confidently claim their theory is accurate is being able to derive force equations from it that accurately predict thrust. Which means they need to build better MET devices to test thrust scaling as predicted. Are there other things Heidi and Woodward need to do that we are not aware of?
1. I presented an exact solution for the force (under suitable, well specified assumptions) and its comparison with experiments.  Prior to the workshop, Prof. Hearn actually independently checked the validity of my solution (which comprises hundreds of terms) by hand !

2. The solution predicts the optimal mass for the tail mass section, in agreement with experiments.  The solution correctly predicts the direction of the force.  The solution predicts the extreme dependence of the amplitude of the force with other parameters in the vicinity of resonance.  The solution also predicts other effects.

3. The solution shows various ways on how it may be possible to increase the force from what has been demonstrated in experiments up to now.

4.  In addition, at the Estes Park Breakthrough Propulsion workshop, a physicist that works with General Relativity presented another derivation of Woodward's equation based on linearization of Einstein's General Relativity. 

5.  In addition, at the Estes Park Breakthrough Propulsion workshop independent experimental measurements of the Mach/Sciama/Wooward effect force were presented from 3 other researchers, in 2 separate continents.

-----

 You will be able to watch my presentation (as well as other presentations) at the Space Studies Institute website: www.ssi.org and ensuing discussion with the participants.

So I am going to beg your forgiveness for what I am about to do...


INNER SPACE GEEK > IT WORKS!!!!!!?
Translation > Given the strong theoretical support. Are you saying that the only thing left is engineering?

Also, I am curious to know if you spent anytime reflecting on the possibility for Mach Effects to "POTENTIALLY" allow the creation of worn holes. I know unlike the thrust term. The Worm hole creation part still has theoretical rough spots. In particular the use of the ADM model of the electron. However, if you have I was curious about your take on that part of Woodward's derivation of Mach Effects.

Probably going to sound a noob question but how can a drive create a worm hole when all along it's been pointed out this is not a warp drive.

Offline Stormbringer

  • Full Member
  • ****
  • Posts: 1340
  • Liked: 239
  • Likes Given: 92
Re: Woodward's effect
« Reply #759 on: 09/25/2016 05:55 pm »
Woodward's derivation of  an equation for mach's principle has two components: a part that is similar to the EM Drive but different in operating detail; and a second part that leads to navigable wormholes.

While currently mucking about with wormholes experimentally is beyond the capabilities of anyone on earth- Woodward  (rightly, I think) says that if the first half of his equation is supported by experiment then the wormhole bit must be correct as well.

So the Woodward drive if proven valid will be highly suggestive that wormhole engineering should eventually work too.

Woodward does not think his Mach drive and the EM drive  operate on the same principle. As of the 1st edition of his book he was skeptical of the EM drive as being anything other than experimental error of some sort. Of course with the cross fertilization that has happened since then (E.G; the SSI conference) he may have changed his opinion on that point. The conference attendees would know if that is still the case or not.
When antigravity is outlawed only outlaws will have antigravity.

Tags:
 

Advertisement NovaTech
Advertisement Northrop Grumman
Advertisement
Advertisement Margaritaville Beach Resort South Padre Island
Advertisement Brady Kenniston
Advertisement NextSpaceflight
Advertisement Nathan Barker Photography
0